Beint í aðalefni
Microsoft
|
Math Solver
Leysa
Æfing
Leika
Efnisatriði
For-Algebra
Meina
Hamur
Stærsti sameiginlegi þátturinn
Minnst Algengar Margfeldi
Röð aðgerða
Broti
Blandað brot
Prime Factorization
Veldisvísir
Róttækur
Algebra
Sameina samsvarandi hugtök
Leysa fyrir breytu
Þáttur
Rýmka
Metið brot
Línulegar jöfnur
Annars stigs jöfnur
Ójöfnuður
Kerfi jöfnur
Fylki
Hornafræði
Einfalda
Meta
Myndrit
Leysa jöfnur
Stærðfræðigreining
Afleiður
Heildir
Takmörk
Algebra inntak
Inntak hornafræði
Stærðfræðigreining Inntak
Fylkisinntak
Leysa
Æfing
Leika
Efnisatriði
For-Algebra
Meina
Hamur
Stærsti sameiginlegi þátturinn
Minnst Algengar Margfeldi
Röð aðgerða
Broti
Blandað brot
Prime Factorization
Veldisvísir
Róttækur
Algebra
Sameina samsvarandi hugtök
Leysa fyrir breytu
Þáttur
Rýmka
Metið brot
Línulegar jöfnur
Annars stigs jöfnur
Ójöfnuður
Kerfi jöfnur
Fylki
Hornafræði
Einfalda
Meta
Myndrit
Leysa jöfnur
Stærðfræðigreining
Afleiður
Heildir
Takmörk
Algebra inntak
Inntak hornafræði
Stærðfræðigreining Inntak
Fylkisinntak
Frum
algebra
hornafræði
Stærðfræðigreining
tölfræði
Fylki
Stafir
Meta
\infty
Spurningakeppni
Limits
5 vandamál svipuð og:
\lim_{ x \rightarrow 0 } \frac{1}{x^2}
Svipuð vandamál úr vefleit
Showing that the \lim_{x\to 0}\frac{1}{x^2} does not exist
https://math.stackexchange.com/q/1579837
Suppose that the limit exists and equals c\in\mathbb{R}. Then for e.g. \epsilon>1 some \delta>0 must exist with \left|x\right|<\delta\implies\left|\frac{1}{x^{2}}-c\right|<1. However, if we ...
Applying L'Hopital's rule to \lim\limits_{x \to 0}\frac{2}{x^2}
https://math.stackexchange.com/questions/502024/applying-lhopitals-rule-to-lim-limits-x-to-0-frac2x2
In order to use the 0/0 case of L'Hospital's rule, we require that both the numerator and the denominator tend to 0 at the appropriate point. The numerator does not tend to 0.
Is this piece-wise function continuous, and why?
https://math.stackexchange.com/questions/2411697/is-this-piece-wise-function-continuous-and-why
If we look at the behaviour as x approaches zero from the right, the function looks like this: \begin{matrix}x & f(x) = \frac{1}{x^2} \\ 1 & 1 \\ 0.1 & 100 \\ 0.01 & 10000 \\ 0.001 & 1000000 \\ 0.0001 & 100000000\end{matrix} ...
Manipulating \lim\limits_{x \to 0}{\frac{\sqrt{x+\sqrt{x}}}{x^n}}
https://math.stackexchange.com/questions/2177214/manipulating-lim-limits-x-to-0-frac-sqrtx-sqrtxxn
If \lim\limits_{x \to 0}{\frac{\sqrt{x+\sqrt{x}}}{x^n}} = c for some c\neq 0, then \lim\limits_{x \to 0}{\frac{x+\sqrt{x}}{x^{2n}}} =c^2. Now, let \sqrt{x}=t. We then wish to find n such ...
Limit of \frac{f'(x)}{g'(x)} & g'(x) \neq 0 in Hypotheses of L'Hospital's rule.
https://math.stackexchange.com/q/110408
When we write things like \lim_{x\to a}h(x) = \lim_{x\to a}H(x) we usually mean "if either limit exists, then they both do and they are equal; if either limit does not exist, then neither limit ...
How do we calculate the Right and Left Hand Limit of 1/x?
https://math.stackexchange.com/questions/762599/how-do-we-calculate-the-right-and-left-hand-limit-of-1-x
\mathbf{Definition} : \boxed{ \lim_{x \to a^+ } f(x) = \infty } means that for all \alpha > 0, there exists \delta > 0 such that if 0<x -a < \delta, then f(x) > \alpha \mathbf{Example} ...
Meira Vörur
Deila
Afrit
Afritað á klemmuspjald
Svipuð vandamál
\lim_{ x \rightarrow 0 } 5
\lim_{ x \rightarrow 0 } 5x
\lim_{ x \rightarrow 0 } \frac{2}{x}
\lim_{ x \rightarrow 0 } \frac{1}{x^2}
Efst á síðu